0

Hallo, ich hab die Aufgabe zu bestimmen, auf welchem Intervall f(X) = x-x^3 konvex, bzw. konkav ist. 
Die Lösung der Turotin lautet: 

 

 

Das verstehe ich nur leider überhaupt nicht. 
Andere Aufgaben konnte ich immer über die zweite Ableitung lösen, hier fällt es mir aber sehr schwer. 

gefragt

 

Habe ich versucht.
Die zweite Ableitung wäre -6x. Ich komme aber zum Ergebnis, dass die Funktion auf dem Intervall (-unendlich,0) konvex und (0,unendlich) konkav wäre.
Ich kann leider nicht nachvollziehen woher die 0,5 aus der Lösung kommen
  ─   thefloatingboat 30.01.2022 um 18:04
Kommentar schreiben
2 Antworten
0
Wann ist denn eine Funktion konkav bzw. konvex?
Diese Antwort melden
geantwortet

Sonstiger Berufsstatus, Punkte: 755

 

Ich hab es so verstanden, dass sie konvex ist, wenn die zweite Ableitung größer als 0 ist, und konkav, wenn sie kleiner als 0 ist.
  ─   thefloatingboat 30.01.2022 um 17:59

1
Richtig. Also zweite Ableitung der Funktion bilden und schauen auf welchen Bereichen sie negativ bzw. positiv ist.   ─   drbau 30.01.2022 um 18:09

Kommentar schreiben

0
Die Bedingung mit der zweiten Ableitung ist nicht die einzige Bedingung für Konvexität/Konkavität. Die Werte kommen übrigens von den Extrempunkten, denn eine Funktion ist auch dann konvex/konkav, wenn die erste Ableitung monoton wachsend/fallend ist. Mit dieser Bedingung bekommt man dann natürlich einen etwas größeren Bereich als nur über die zweite Ableitung. Die Grafik verdeutlicht das ganz gut.

Edit: Hier steht natürlich Quatsch. Hab mich von dem Bild jetzt zu sehr verwirren lassen. Die Funktion ist natürlich nur auf dem Bereich $(-\infty; 0]$ konvex bzw. auf dem Bereich $[0;\infty)$ konkav. Damit sind dann die Bedingung an die zweite Ableitung bzw. die Monotoniebedingung der ersten Ableitung erfüllt. Das Bild ist da etwas trügerisch, denn die Bereiche gehen nicht etwa bis zu den Extrempunkten und konvex bedeutet ja bspw., dass ich zwei Punkte aus dem Bereich mit einer Strecke verbinden kann und der gesamte Funktionsgraph unterhalb dieser Strecke liegt. Das erscheint um den Wendepunkte herum auch erst einmal so auszusehen. Wenn man sich die Funktion aber mal zeichnen lässt und stark reinzoomt, sieht man, dass die Konvexität etwa im Bereich $[-0{,}5;0]$ nicht gilt.
Diese Antwort melden
geantwortet

Selbstständig, Punkte: 30.55K

 

Alles klar, es macht schon Sinn. Jetzt erkenne ich es auch an der Grafik.
Wenn also gefragt ist, auf welchem Intervall es konvex/konkav ist, schaue ich mir die erste Ableitung an; wenn aber nur gefragt würde, an welchem Punkt es von konvex auf konkav umschwenkt, dann reicht mir die zweite Ableitung?
  ─   thefloatingboat 30.01.2022 um 20:32

1
Wenn die erste Ableitung der Funktion f monton fallend/wachsend ist, dann bedeutet dies doch f''(x) > 0 bzw. f''(x) < 0. Eine Veranschaulichung, so wie ich sie kenne, ist folgende: eine Funktion ist in einem Intervall \(I=(a, b)\) konvex, wenn alle Funktionswerte von f innerhalb des Intervalls \(I\) unterhalb der über die Intervallgrenzen bestimmten Sekante liegen. Hatte die vorgegebene Lösung leider erst nicht gesehen, denn ich wäre hier zu dem gleichen Schluss gekommen, wie der/die Fragestellende. Also sorry, wenn ich doch nicht helfen konnte. Vielleicht fragst du deine Tutorin oder cauchy oder jemand anderes aus dem Forum kann hierzu noch etwas sagen.   ─   drbau 31.01.2022 um 11:59

Vielen Dank. Die Lösung der Tutorin hat mich fast verzweifeln lassen, aber wenn sie alle zu dem selben Ergebnis wie ich kommen, bin ich beruhigt.   ─   thefloatingboat 01.02.2022 um 11:20

Wenn für dich die Fragestellung nun geklärt ist, kannst du die Frage gerne noch als erledigt kennzeichnen.   ─   drbau 02.02.2022 um 17:04

Leider scheint diese Antwort Unstimmigkeiten zu enthalten und muss korrigiert werden. Cauchy wurde bereits informiert.